Which inequality represents all the solutions of 8(6x-7) <5(9x - 4)?
O A.
OB.
OOO
O C.
OD.
X> 12
x < 12
x> 20
x < 20

Answers

Answer 1

Hello,

8(6x-7) <5(9x - 4)

48x - 56 < 45x - 20

48x - 45x < -20 + 56

3x < 36

x < 36/3

x < 12


Related Questions

millimeters =equals 15centimeters 4millimeters

Answers

This shows that 154mm is equivalent to 15centimeters 4millimeters

Conversion of units

Units are measure of a quantity. From the given question, we are to convert 15cm4mm to millimeters

Since 10mm =1cm, hence;

15cm4mm = (15 * 10) + 4

15cm4mm = 150+ 4

15cm4mm = 154mm

This shows that 154mm is equivalent to1 5centimeters 4millimeters

Learn more on unit conversion here: https://brainly.com/question/3371568

#SPJ1

The endpoints of the
directed line segment AB
are A (1,7) and B(5, 15).
Find the coordinates of point
P along AB so that the ratio
of AP to PB is 3 to 1.

Answers

The coordinates of point P along AB so that the ratio of AP to PB is 3 to 1 is (4,13).

What are coordinates?

A coordinate system in geometry is a system that employs one or more integers, or coordinates, to define the position of points or other geometric components on a manifold such as Euclidean space.

Given that the ratio of the two lines is 3:1, therefore, we can write,

m:n = 3:1

Also, the coordinate of the endpoints of the line are,

A = (x₁, y₁) = (1, 7)

B = (x₂, y₂) = (5, 15)

Now, Using the Section formula the coordinate of the point P can be written as,

[tex]P = (\dfrac{mx_2+nx_1}{m+n}\ , \dfrac{my_2+yx_1}{m+n})\\\\P = (\dfrac{3(5)+1(1)}{4}\ , \dfrac{3(15)+1(7)}{4})\\\\P = (4, 13)[/tex]

Hence, the coordinates of point P along AB so that the ratio of AP to PB is 3 to 1 is (4,13).

Learn more about Coordinates:

https://brainly.com/question/23450276

#SPJ1

Select
Select the correct answer from each drop-down menu.
Which system of Inequalities does the graph represent? Which test point satisfies both of the inequalities in that system?

Answers

The inequality equation of the lines are given as 4x – y ≤ 4 and 2x – 2y ≤ 3. Then the correct option is A.

What is inequality?

Inequality is defined as an equation that does not contain an equal sign. Inequality is a term that describes a statement's relative size and can be used to compare these two claims.

The graph is given below.

From the graph, equation of the lines are given as

4x - y = 4 and 2x - 2y = 3

But for the inequality is shown in graph. Then the inequality equation of the lines are given as

4x – y ≤ 4 and 2x – 2y ≤ 3

Then the correct option is A.

More about the inequality link is given below.

https://brainly.com/question/19491153

#SPJ1

A simple random sample of size individuals who are currently employed is asked if they work at home at least once per week. Of the employed individuals​ surveyed, responded that they did work at home at least once per week. Construct a​ 99% confidence interval for the population proportion of employed individuals who work at home at least once per week.

Answers

The confidence interval of employes individuals from home

Construction of hypothesis:

0.10 - 0.047 < p < 0.10 + 0.047

A simple random sample size n = 250 . Of the 250 employed individuals ​surveyed,42 responded that they did work home a minimum of once per week.

Construct 99% confidence interval for population

For proportion : 42 / 250 = 1/10 = 0.16

Mean = 2.5 * sqrt [ 0.1 * 0.9 / 250]

= 2.5 * 0.01

= 0.47

Construction of hypothesis:

0.10 - 0.047 < p < 0.10 + 0.047

To know more about confidence interval

https://brainly.com/question/24131141

#SPJ4

Area= in2
Help me please!! Thank you so much

Answers

Answer:

60 in²

Step-by-step explanation:

Area of a Regular Polygon

  [tex]\textsf{A}=\dfrac{n\:s\:a}{2}[/tex]

where:

A = arean = number of sidess = length of one sidea = apothem (the line drawn from the center of any polygon to the midpoint of one of the sides)

From inspection of the diagram:

n = 5s = 6 ina = 4 in

Substitute the values into the formula and solve for A:

[tex]\implies \sf A = \dfrac{5 \times 6 \times 4}{2}[/tex]

[tex]\implies \sf A = \dfrac{120}{2}[/tex]

[tex]\implies \sf A = 60\:in^2[/tex]

Therefore, the area of the given regular pentagon IJKLM is 60 in².

[tex]\huge\text{Hey there!}[/tex]

[tex]\huge\textbf{First thing, let's get the formula for}\\\huge\textbf{the regular pentagon/polygon.}[/tex]

[tex]\mathbf{\dfrac{1}{2}\times \boxed{a}pothem\times\boxed{p}erimeter = {\boxed{a}rea}}[/tex]

[tex]\huge\textbf{Or we could simply say....}[/tex]

[tex]\mathbf{\dfrac{\boxed{n}umber\ of\ sides\ you\ have \times \boxed{s}ide \ length\ on\ one\ side\times \boxed{a}pothem}{2} = \boxed{a}rea}[/tex]

[tex]\huge\textbf{Let's plug the pieces to the equation,}\\\huge\textbf{since we have that information out the}\\\huge\textbf{way.}[/tex]

[tex]\bullet\mathbf{\ Number\ of\ sides \rightarrow \boxed{\bf 5}}[/tex]

[tex]\bullet\mathbf{\ Side\ length\ of\ one\ side \rightarrow \boxed{\bf 6\ inches}}[/tex]

[tex]\bullet\mathbf{\ Apothem \rightarrow \boxed{\bf 4\ inches}}[/tex]

[tex]\huge\textbf{Making the equation for you:}[/tex]

[tex]\mathbf{\dfrac{5\times 6 \times 4}{2} = \boxed{a}rea}[/tex]

[tex]\huge\textbf{Solving for the result:}[/tex]

[tex]\mathbf{\dfrac{5\times 6 \times 4}{2} = \boxed{a}rea}[/tex]

[tex]\mathbf{\dfrac{30\times 4}{2} = \boxed{a}rea}[/tex]

[tex]\mathbf{\dfrac{120}{2} =\boxed{a}rea}[/tex]

[tex]\mathbf{\dfrac{120\div2}{2\div2} = \boxed{a}rea}[/tex]

[tex]\mathbf{\dfrac{60}{1} = \boxed{a}rea}[/tex]

[tex]\mathbf{60\div1 = \boxed{a}rea}[/tex]

[tex]\mathbf{60 = \boxed{a}rea}[/tex]

[tex]\huge\textbf{Met the result....}[/tex]

[tex]\huge\text{Thus, your answer should be: \boxed{\mathsf{60\ in^2}}}\huge\checkmark[/tex]

[tex]\huge\text{Good luck on your assingment \& enjoy your day!}[/tex]

~[tex]\frak{Amphitrite1040:)}[/tex]

ASAP PLEASE (this is like five questions in one) GIVING 15 POINTS!!!
3. find x
4. find x
7. find y
6. find x
8. find y

Answers

Answer:

D

21√2/2

C

C

C

Step-by-step explanation:

For all problems, the ratios of the lengths of the sides are:

30-60-90 triangle 1 : √3 : 2

45-45-90 triangle 1 : √2 : √2

Problem 3:

30-60-90 triangle at left

Hypotenuse = 11

Short leg = 11/2

Long leg = 11/2 × √3

45-45-90 triangle at right

x = 11/2 × √3 × √2 = 11√6/2

Answer: D

Problem 4:

30-60-90 triangle at top

Hypotenuse = 7√3

Short leg = 7√3/2

Long leg = 7√3/2 × √3 = 7 × 3/2 = 21/2

45-45-90 triangle at right

x = 21/2 × √2 = 21√2/2

Answer: 21√2/2

Problem 7:

30-60-90 triangle

Long leg = 4√3

Short leg = y = 4√2/2 = 2√2

Answer: C.

Problem 6:

30-60-90 triangle at left

Hypotenuse = 12√6

Short leg = 12√6/2 = 6√6

Long leg = 6√6 × √3 = 6√2√3√3 = 18√2

45-45-90 triangle at right

x = 18√2/√2 = 18

Answer: C.

Problem 8:

45-45-90 triangle

Hypotenuse = 2

Leg = y = 2/√2 = 2√2/(√2√2) = √2

Answer: C.

Use the Law of Cosines to find the specified missing value. Approximate your answer to the nearest tenth.

C=60, a=120ft, b=66ft

Answers

Answer:

c=104.1 ft

Step-by-step explanation:

Law of Cosines

c^2 = 66^2 + 120^2 - 2 (66)(120)cos 60

c = 104.1 ft

PLEASE QUICKLY!!!

The length of a rectangular frame is represented by the expression 3x + 6, and the width of the rectangular frame is represented by the expression 3x + 4. Write an equation to solve for the width of a rectangular frame that has a total area of 120 square inches.

Answers

The width of the rectangle is 10 inches

How to determine the width?

The given parameters are:

Length = 3x + 6

Width = 3x + 4

Area = 120

The area of a rectangle is

Area = Length * Width

So, we have:

(3x + 6) *(3x + 4) = 120

Expand

9x^2 + 12x + 18x + 24 = 120

Evaluate the like terms

9x^2 + 30x - 96 = 0

Using a graphing calculator, we have:

x = 2

Substitute x = 2 in Width = 3x + 4

Width = 3 * 2 + 4

Evaluate

Width = 10

Hence, the width of the rectangle is 10 inches

Read more about areas at:

https://brainly.com/question/11952845

#SPJ1

If f(x) is a linear function, what is the value of n?
X
-4
-1
n
2
O 4
05
09
f(x)
-25
-10
20

Answers

Answer:

3rd option : 5

Step-by-step explanation:

https://brainly.com/question/8346797

its a slope question

f(x) is y

y2-y1/x2-x1

-10+25/-1+4

15/3

5

Since we know that the equation of a line in slope-intercept form is: , where, m= Slope and b= y-intercept.

Upon substituting m = 5 in slope-intercept form of equation we will get,

Now let us find y-intercept by substituting coordinates of one point in our given line.

So the equation of given line will be,

Now let us find value of n by substituting x = n and y = 20 in our equation.

Therefore, the value of n is 5.

x^(2)-5x+6=(x+h)^(2)+k solve for k

Answers

The value of k is  [tex]-h^2 -2xh -5x + 6[/tex]

How to solve the expression

Given;

x^(2)-5x+6=(x+h)^(2)+k

Expand the expression on the right side, we have

[tex]x^2 - 5x + 6 = (x+ h) (x+ h) + k[/tex]

[tex]x^2 -5x + 6 = x^2 +xh + xh + h^2 +k[/tex]

Collect like terms

[tex]x^2 -5x + 6 = x^2 + 2xh + h^2 + k[/tex]

[tex]x^2 -5x + 6 -x^2 - 2xh -h^2 = k[/tex]

We then have

[tex]-5x + 6 -2xh -h^2 = k[/tex]

[tex]k = -h^2 -2xh -5x + 6[/tex]

Thus, we have k as [tex]-h^2 -2xh -5x + 6[/tex]

Learn more about algebraic expression here:

https://brainly.com/question/723406

#SPJ1

A counter in Lamar's kitchen is 1 yard wide and 2 yards long. Lamar wants to replace the counter with a granite countertop, which would cost $204.63 per square yard. How much would the granite countertop cost?

Answers

Answer:

Step-by-step explanation:

Comment

Find the number of square yards

Area = L * w

L = 2

w = 1

Area = 2 * 1

Area = 2

Now for the counter top

1 sq yard costs 204.63

2 sq yards cost 2 * 204.63

2 sq yards cost 409.26

Answer: 409.26


please answer algebra question. will mark brainliest
answers:
1. 9
2. 12
3. 18
4. 36 sq cm

Answers

Answer:

9 sq cm

Step-by-step explanation:

If you join the two triangles it will become a square of side 3cm.

3cm * 3cm = 9cm squared.

Hope you have a nice day :)

Any questions please comment.

Which equation has a graph that is a parabola with a vertex at (–2, 0)?

Answers

The equation of the parabola which has a vertex at (-2,0) is; y = x² +4x +4.

What is the equation of the parabola with vertex (-2,0)?

From the task content, it follows that the vertex of the of the parabola described by the graph in discuss is given by the point; (-2,0).

Hence, the vertex form of the equation is;

y = (x-(-2))² +0

Hence, the equation is; y = x² +4x +4.

Read more on the equation of a parabola;

https://brainly.com/question/4148030

#SPJ1

Answer: B

Step-by-step explanation:

Edge 2023

Find side x, giving your answer to 1 decimal place.
7 cm
81°
X
40°

Answers

In the given diagram, the value of the side labelled x is 10.8 cm

Law of sines

From the question, we are to determine the value of side x

Using the law of sines

In any given triangle XYZ,

[tex]\frac{x}{sinX}=\frac{y}{sinY}=\frac{z}{sinZ}[/tex]

Thus,

In the given triangle,

[tex]\frac{x}{sin81^\circ} =\frac{7}{sin40^\circ}[/tex]

∴ [tex]x=\frac{7 \times sin81^\circ}{sin40^\circ}[/tex]

x = 10.75599

x ≈ 10.8 cm

Hence, the value of the side labelled x is 10.8 cm

Learn more on Law of sines here: https://brainly.com/question/2807639

#SPJ1

4(x + 4) + 6

Simplify algebraic expressions

Answers

Answer:

4x+22

Step-by-step explanation:

distribute 4(x+4)

it would become 4x+16+6

16+6 = 22

4x+22

The total surface area of a closed cylinder is
5000 cm2
find the dimensions of the cylinder that
maximize its volume and state this maximum
volume.
verify that it is a maximum point.

Answers

The maximum volume of the cylinder is 27147.355 at the maximum point  [tex]r = \frac{50}{\sqrt{3} }[/tex] .

How do you find the maximum volume of the cylinder?

The formula for the volume of the cylinder v = [tex]\pi[/tex][tex]r^{2}[/tex]h, To find the maximum volume of the cylinder we apply the condition of maxima  [tex]\frac{\mathrm{d} v}{\mathrm{d} r}[/tex] = 0.

Let r cm be the radius and h cm be the height of the closed cylinder.

Then, Total surface area of the cylinder = [tex]2\pi r(r+h)[/tex]

                                                        [tex]5000[/tex] = [tex]2\pi r^{2} +2\pi rh[/tex]

                                                             [tex]h[/tex] =   [tex]\frac{5000-2\pi r^{2} }{2\pi rh}[/tex] .............(1)

Volume of the cylinder v = [tex]\pi r^{2} h[/tex]

Substitute the value of [tex]h[/tex] in the above equation

[tex]\Rightarrow[/tex]                                       = [tex]\pi r^{2}[/tex] × [tex]\left [ \frac{5000-2\pi r^{2}}{2\pi r} \right ][/tex]

[tex]\Rightarrow[/tex]                                      = [tex]\frac{r}{2}[/tex] ×  [tex](5000-2\pi r^{2} )[/tex]

[tex]\Rightarrow[/tex]                                    v = [tex]2500r-\pi r^{3}[/tex] ..............(2)

Now, for the maximum volume of the cylinder  [tex]\frac{\mathrm{d} v}{\mathrm{d} r}[/tex] = 0

[tex]\Rightarrow[/tex]                                                            [tex]\frac{\mathrm{d} (2500r-\pi r^{3})}{\mathrm{d} r} = 0[/tex]

[tex]\Rightarrow[/tex]                                                                      [tex]3\pi r^{2} = 2500[/tex]

[tex]\Rightarrow[/tex]                                                                          [tex]r^{2} = \frac{2500}{3\pi }[/tex]

[tex]\Rightarrow[/tex]                                                                          [tex]r = \frac{50}{\sqrt{3\pi } }[/tex]

Volume is maximum for  [tex]r = \frac{50}{\sqrt{3\pi } }[/tex]  

Then, v = [tex]2500r-\pi r^{3}[/tex]

[tex]\Rightarrow[/tex]              =  [tex]2500 \frac{50}{\sqrt{3\pi } } -\pi (\frac{50}{\sqrt{3\pi } } )^{3}[/tex]                              

[tex]\Rightarrow[/tex]             = [tex]\frac{125000}{\sqrt{3\pi } } - \frac{125000}{3\sqrt{3\pi } }[/tex]

[tex]\Rightarrow[/tex]             = [tex]\frac{125000}{\sqrt{3\pi } }[/tex]×[tex]\frac{2}{3}[/tex]

[tex]\Rightarrow[/tex]             = [tex]\frac{250000}{3\sqrt{3\pi } }[/tex]                                                              

[tex]\Rightarrow[/tex]          v = 27147.355

Hence, The maximum volume of the cylinder is 27147.355 at the maximum point  [tex]r = \frac{50}{\sqrt{3} }[/tex] .                                                

To learn more about total surface area and volume of the cylinder from the given link

https://brainly.com/question/16095729

#SPJ4                                                                                   

Please help!! Which of the following graphs shows a parabola with a vertex of (1,-9) and solutions of (-2,0) and (4,0)?

Answers

The graphs show a parabola with a vertex that has Roots are -2 and 4.

We have given that,

vertex of (1,-9) and solutions of (-2,0) and (4,0).

What is the formula for the roots?

y=k*(x+2)(x-4)

Vertex = (1,-9) is a point of the parabola

-9=k*(1+2)(1-4) implies k=1

Equation of the parabola is y=(x+2)(x-4).

The graphs show a parabola with a vertex that has Roots are -2 and 4.

To learn more about the parabola visit:

https://brainly.com/question/4148030

#SPJ1

PLEASE HURRY!!!!!!!
The manager of a local cable TV company wants to know which of its programs are most popular. Which population should the manager survey to best answer this question?

A. Employees of the cable company.
B. Subscribers to the cable company.
C. Adults listed in the town’s voter registration lists.
D. People who complete a survey on the company’s Web site.

Answers

Answer:

B

Step-by-step explanation:

cable subscribers

The table below shows the x variable for a table. Choose a corresponding y table that will create a direct variation.

Answers

Answer:

table A

Step-by-step explanation:

A direct variation can be expressed as: [tex]\frac{y}{x}=k[/tex]. The y/x is equal to a constant value k. Multiply both sides by x you get: [tex]y=kx[/tex]. So as x increases by 1, y increases by some constant value. So to find which one will create a direct variation, see which of the tables has a constant y/x

Table A:
   7/2 = 3.5

   14/4 =3.5

   17.5/5=3.5

   21/6 = 3.5

   this has a constant y/x so it is a direct variation

Table B:

   7/2 = 3.5

   14/4 = 3.5

   21 / 5 = 4.2

   this is not a direct variation, it doesn't have a constant y/x

table C:

   7 / 2 = 3.5

   14 / 4 = 3.5

   15/5 = 3

   This is not a direct variation, it doesn't have a constant y/x

table d:
   7/2 = 3.5

   14/4 =3.5

   17/5=3.4

   This is not a direction variation, it doesn't have a constant y/x

Bethany wants to estimate the value of (4.296 times 10 Superscript 11) (1.8614 times 10 Superscript negative 14). Which statement about the estimate is true?
The value will be greater than 1 because 4.296 times 10 Superscript 11 is a very large number and multiplication always increases the size of a number.
The value will be greater than 1 because 4 times 2 = 8 and 8 is larger than 1.
The value will be less than 1 because (4.296 times 10 Superscript 11) (1.8614 times 10 Superscript negative 14) will be a negative number. All negative numbers are less than one.
The number will be less than 1 because when adding the exponents, 11 + (negative 14) = negative 3, and a number in scientific notation with a negative exponent is less than 1.

Answers

The statement that is true about the estimate is that the number will be less than 1 because when adding the exponents, 11 + (negative 14) = negative 3, and a number in scientific notation with a negative exponent is less than 1 (option D).

How to estimate an exponential function?

According to this question, Bethany wants to estimate the value of (4.296 times 10 Superscript 11) (1.8614 times 10 Superscript negative 14).

This expression can be mathematically written as:

(4.296 × 10¹¹) (1.8614 × 10-¹⁴)

The result of this expression is 7.996 × 10-³, which is value less than 1.

Therefore, the number will be less than 1 because when adding the exponents, 11 + (negative 14) = negative 3, and a number in scientific notation with a negative exponent is less than 1.

Learn more about exponentiation at: https://brainly.com/question/11487261

#SPJ1

Twenty children were surveyed about the number of hours they play outside in 1 week. The results are shown below.

6, 5, 4, 3, 0, 7, 1, 5, 4, 4, 3, 2, 2, 3, 2, 4, 3, 4, 0, 7

Complete and upload the frequency table below and create a histogram to display this data. Make sure there is only 4 intervals.

Answers

The data can be shown in the frequency table with only 4 intervals and the histogram is shown in the picture.

What is a histogram?

It is defined as the depiction of numerical data in a graph using a bar with no space. The histogram depicts the approximate distribution of the data.

We have data:

6, 5, 4, 3, 0, 7, 1, 5, 4, 4, 3, 2, 2, 3, 2, 4, 3, 4, 0, 7

Intervals   Frequency

 0 - 3     6

 3 - 6     11

 6 - 9     3

 9 - 12     0

The histogram is shown in the attached picture.

Thus, the data can be shown in the frequency table with only 4 intervals and the histogram is shown in the picture.

Learn more about the histogram here:

brainly.com/question/16819077

#SPJ1

please help me asap thank you so so much u are so amazing!!

Answers

Answer:

7.76

Step-by-step explanation:

0.22×20=4.4

3.36(base amount ,average of the others)+4.4=7.76

(LOOK AT IMAGE)
Line CDE is a tangent to the circle below.

Work out the size of angle x and the size of angle y. ​

Answers

[tex]x=57^{\circ}[/tex] (alternate segment theorem)

[tex]y=67^{\circ}[/tex] (alternate segment theorem)

What is the gradient of the graph shown?

Answers

Answer:

gradient = [tex]\frac{1}{3}[/tex]

Step-by-step explanation:

calculate the gradient m using the gradient formula

m = [tex]\frac{y_{2}-y_{1} }{x_{2}-x_{1} }[/tex]

with (x₁, y₁ ) = (- 6, 0) and (x₂, y₂ ) = (0, 2) ← 2 points on the line

m = [tex]\frac{2-0}{0-(-6)}[/tex] = [tex]\frac{2}{0+6}[/tex] = [tex]\frac{2}{6}[/tex] = [tex]\frac{1}{3}[/tex]

is x + 3 a factor of f(x)=x^4+6x^3+5x^2-15x-2

Answers

x+3 is not a factor of f(x)=x^4+6x^3+5x^2-15x-2 because there is a remainder of 7 when we use the long division method to divide x+3 with the given function f(x).

What are the factors of a quadratic equation?

The factors of a quadratic equation are the simplified form of the quadratic equation, that if multiplied together, results in the original quadratic equation.

Given that:

f(x)=x^4+6x^3+5x^2-15x-2

[tex]\mathbf{=\dfrac{x^4+6x^3+5x^2-15x-2}{x+3} }[/tex]

[tex]\mathbf{=x^3+3x^2-4x-3+\dfrac{7}{x+3} }[/tex]

x+3 is not a factor of f(x)=x^4+6x^3+5x^2-15x-2 because there is a remainder of 7 when we use the long division method to divide x+3 with the given function f(x).

Learn more about finding the factors of a quadratic equation here:

https://brainly.com/question/3400933

#SPJ1

What is the pattern in the values as the exponents increase?

Powers of 2
Value
2 Superscript negative 5
StartFraction 1 Over 32 EndFraction
2 Superscript negative 4
StartFraction 1 Over 16 EndFraction
2 Superscript negative 3
StartFraction 1 Over 8 EndFraction
2 Superscript negative 2
One-fourth
2 Superscript negative 1
One-half
2 Superscript 0
1

add StartFraction 1 Over 16 EndFraction to the previous value
subtract StartFraction 1 Over 16 EndFraction from the previous value
divide the previous value by 2
multiply the previous value by 2

Answers

The pattern in the values as the exponents increase is multiply the previous value by 2.

What is exponents and powers?

Exponents and powers are ways used to represent very large numbers or very small numbers in a simplified manner.

Given:

Powers of 2      Values

[tex]2^{-5}[/tex]                         1/32

[tex]2^{-4}[/tex]                           1/16

[tex]2^{-3}[/tex]                           1/8

[tex]2^{-2}[/tex]                           1/4

[tex]2^{-1}[/tex]                           1/2

As, from the given data we can see that negative powers of 2 reducing by 1.

as, 2^-5 * 2= 2^-4

Hence, multiply the previous value by 2 to obtain the next value.

Learn more about exponents and powers here:

https://brainly.com/question/16649216

#SPJ1

A right triangle has side lengths AC = 7 inches, BC = 24
inches, and AB = 25 inches.

What are the measures of the angles in triangle ABC?

A) mZA= 46.2°, mZB=43.8°, mZC=90°
B) mZA= 73.0°, mZB= 17.0°, mZC = 90°
C) mZA 73.7°, mZB= 16.3°, mZC = 90°
D) mZA 74.4°, mZB= 15.6°, mZC = 90°

Answers

Therefore, the measures of the angles in triangle ABC is as follows:

m∠A 73.7°, m∠B = 16.3°, m∠C = 90°

How to find angles of right triangle?

The measure of the angles in the right triangle can be found as follows:

Using trigonometric ratios,

sin ∅ = opposite / hypotenuse

sin ∅ = 7 / 25

∅ = sin ⁻¹ 0.28

∅ = 16.2602047083

∅ = 16.3°

Hence,

m∠B = 16.3°

m∠C = 90°

m∠A = 180 - 90 - 16.3 = 73.7°

learn more on right triangle here: https://brainly.com/question/27899600

#SPJ1

Given quadrilateral ABCD and its image under a dilation centered at the origin, find the scale factor. (note: orange is original image)

Answers

The scale factor of the image under a dilation centered at the origin is 2.

What is a transformation?

Transformation is the movement of a point from its initial location to a new location. Types of transformation are reflection, translation, rotation and dilation.

Translation is the movement of a point either up, down, left or right on the coordinate plane.

Let us assume that quadrilateral ABCD have vertices at A(2, 1) while its image has vertices at A'(4, 2), hence:

Scale factor = 4/2 = 2/1 = 2

The scale factor of the image under a dilation centered at the origin is 2.

Find out more on transformation at: https://brainly.com/question/4289712

#SPJ1

A plane flying horizontally at an altitude of 1 mile and a speed of 570 mi/h passes directly over a radar station. Find the rate at which the distance from the plane to the station is increasing when it has a total distance of 3 miles away from the station. (Round your answer to the nearest whole number.)

Answers

The rate at which the distance from the plane to the station increases is 537 miles/hour. Computed using Pythagoras Theorem and Differentiation.

We assume the station to be at point A.

The plane when directly over the station, takes point C.

As it is flying horizontally continuously at an altitude of 1 mile, AC = b = 1 mile.

The plane flies continuously, and when it is at a distance of 3 miles from the station, it is at point B.

The distance between the plane and the station of 3 miles can be shown as AB = c = 3 miles.

The movement of the plane from C to B can be shown as BC = a.

Now, from the given case, we get a right triangle ABC.

By Pythagoras' Theorem,

a² + b² = c² ... (i).

or, a² + 1² = 3²,

or, a² = 9 - 1 = 8,

or, a = 2√2 miles.

We are told that the plane is moving at a speed of 570 miles/hour.

This can be shown as, da/dt = 570 miles/hour.

As the plane is moving horizontally, the vertical distance between the plane and the station doesn't change, that is, db/dt = 0.

In the question, we are asked to find the rate at which the distance from the plane to the station is increasing, that is, we are asked to find dc/dt.

Differentiating (i), with respect to time t, we get:

2a(da/dt) + 2b(db/dt) = 2c(dc/dt).

Substituting values of a = 2√2 miles, b = 1 mile, c = 3 miles, da/dt = 570 miles/hour, and db/dt = 0, we get:

2(2√2)(570) + 2(1)(0) = 2(3)(dc/dt),

or, 2280√2 + 0 = 6(dc/dt),

or, dc/dt = 380√2 miles/hour = 537.40 miles/hour ≈ 537 miles/hour.

Thus, the rate at which the distance from the plane to the station increases is 537 miles/hour. Computed using Pythagoras Theorem and Differentiation.

Learn more about the Pythagoras Theorem and Differentiation at

https://brainly.com/question/15291822

#SPJ4

HELP FAST! What is the equation of the line graphed?
(y=mx+b)

Answers

Answer y=3x + 5
Step by step

Start by finding two points on the graph
(2,1) and (3,4)
Find slope (m)

(y2 - y1) over (x2 - x1)
(4-1) over (3-2)
3/1 or the slope of 3 (m)
The y intercept is 5 (b) where the line crosses the y
So y= 3x + 5
Other Questions
Select the correct answer.Which of the following best describesone of the reasons Kennedy won the first debate against Nixon in 1960?O A. Kennedy was tanned, rested, and relaxed.O B. Nixon was unprepared for the debate and stumbled over his words.O c. Nixon's accent made him difficult to understand for many listeners.O D. Kennedy knew what the questionsquestions were going to be ahead of time. You are a loan officer at the West Elm Savings and Loan. Mr. and Mrs. Brady are in your office to apply for a mortgage loan on a house they want to buy. The house has a market value of $160,000. Your bank requires 1/5 of the market value as a down payment.(a)What is the amount (in $) of the down payment? $(b)What is the amount (in $) of the mortgage for which the Bradys are applying? $(c)Your bank offers the Bradys a 30-year mortgage with a rate of 5%. At that rate, the monthly payments for principal and interest on the loan will be $5.37 for every $1,000 financed. What is the amount (in $) of the principal and interest portion of the Bradys' monthly payment? $(d)What is the total amount (in $) of interest that will be paid over the life of the loan? $(e)Your bank also requires that the monthly mortgage payments include property tax and homeowners insurance payments. If the property tax is $1,710 per year and the property insurance is $1,458 per year, what is the total monthly payment (in $) for PITI (principal, interest, taxes, and insurance)? $(f)To qualify for the loan, bank rules state that mortgage payments cannot exceed 14 of the combined monthly income of the family. If the Bradys earn $3,750 per month, will they qualify for this loan?Yes, they will http://qualify.No, they will not qualify.(g)What monthly income (in $) would be required to qualify for this size mortgage payment? $ question on the image 5. Why is exercise an important part of a weight-lossprogram? Why is exercise also important forgaining weight? Ashlyn's research focuses on how the city where she lives interacts with the natural environment. This field of study is called __________. environmental sociology social ecology new ecological paradigm explain four ways in which faulting influences the drainage system What is the solution to the equation1/4x-1/8=7/8+1/2x? If you were to deliver a speech advocating a new health insurance policy, it would be most helpful to begin by establishing that there is a(n) ______ for it. If a linear function contains the pairs (5,7) and (9,11), which pairs should have the reciprocal function to this one?Please choose an answer.a.(7.5) and (11.9)b.(5.7) and (11.9)c.(5.7) and (9.11)d.(7.5) and (9.11) Which table or graph shows the value of y going up as the value of x goes up?O A. x y16OB.2349896SHSN14 PREVIOUS121824 Placing barcodes on multiple sides of products and not bagging groceries are examples of ____________________ which allow Aldi to maximize efficiency. Eastern European folk music was influenced primarily by a. African culture b. Russian culture c. Moslem culture d. Celtic culture graph absolute value of x-1 A door delivery florist wishes to estimate the proportion of people in his city that will purchase his flowers. Suppose the true proportion is 0.07. If 289 are sampled, what is the probability that the sample proportion will differ from the population proportion by more than 0.03 Annie's team has just finished a major project and the team has, after a long time, got some free time on hand. however, in the last couple of days, annie has noticed that few of the team members are immersed in online social networking sites during office hours. what is the least effective responsive? a textbook company ships its books in boxes that can hold 30 pounds. one particular order calls for a shipment of 800 books. if each book weighs 1.4 kilograms, how many boxes will be required to fulfill this order PLEASE ANSWER ASAP: Miriam bought 64 pounds of popcorn she kept 1/4 for her family then gave 1/8 of the remainder to her club and distributed the rest equally among 8 friends. How many ounces did each friend get? homework! This is a stats problem Ive been struggling with and need help thanks!!! Find K such that the number 23K10 isdivisible by 2, 3, and 52021 Price of article = $315.50Down payment = $31.55Monthly payment amount = $16.50Duration of payments = 20 months Interest(I)=2ycm(n+1)True annual interest rate to the nearest tenth=